How do u implicitly differentiate (xy)^1/2

  • Thread starter Thread starter engineer_dave
  • Start date Start date
  • Tags Tags
    Differentiate
Click For Summary
SUMMARY

The discussion centers on the implicit differentiation of the expression (xy)^(1/2). The correct approach involves applying the chain rule, specifically using the formula (1/2)(xy)^(-1/2) multiplied by the derivative of xy with respect to x, which includes dy/dx. Participants clarify that implicit differentiation requires an equation to differentiate, and the initial attempt does not constitute implicit differentiation as no equation is provided.

PREREQUISITES
  • Understanding of implicit differentiation
  • Familiarity with the chain rule in calculus
  • Knowledge of derivatives of products (product rule)
  • Basic algebraic manipulation skills
NEXT STEPS
  • Study the chain rule in depth
  • Learn about implicit differentiation with examples
  • Review the product rule for differentiation
  • Practice solving implicit differentiation problems
USEFUL FOR

Students studying calculus, particularly those learning about differentiation techniques, and educators seeking to clarify concepts of implicit differentiation.

engineer_dave
Messages
35
Reaction score
0

Homework Statement



How do u implicitly differentiate (xy)^1/2

Homework Equations





The Attempt at a Solution



Would it be 1/2(xy)^-1/2 multiplied by 1 and dy/dx. Thanks
 
Physics news on Phys.org
It looks like you are trying to use the chain rule so you are on the right track. What's d/dx of xy? BTW, so far, this isn't implicit differentiation.
 
what do u mean it isn't implicit differentiation?
 
It's just differentiation. Implicit differentiation is where you take d/dx of an equation and then solve for dy/dx. There's no equation yet. And you haven't answered my question.
 
Question: A clock's minute hand has length 4 and its hour hand has length 3. What is the distance between the tips at the moment when it is increasing most rapidly?(Putnam Exam Question) Answer: Making assumption that both the hands moves at constant angular velocities, the answer is ## \sqrt{7} .## But don't you think this assumption is somewhat doubtful and wrong?

Similar threads

  • · Replies 2 ·
Replies
2
Views
2K
  • · Replies 25 ·
Replies
25
Views
2K
  • · Replies 7 ·
Replies
7
Views
2K
  • · Replies 15 ·
Replies
15
Views
2K
  • · Replies 32 ·
2
Replies
32
Views
4K
  • · Replies 20 ·
Replies
20
Views
2K
  • · Replies 6 ·
Replies
6
Views
4K
  • · Replies 6 ·
Replies
6
Views
2K
  • · Replies 3 ·
Replies
3
Views
2K
  • · Replies 5 ·
Replies
5
Views
3K